Đến nội dung

Hình ảnh

Đề Thi VMO năm 2017


  • Please log in to reply
Chủ đề này có 73 trả lời

#21
royal1534

royal1534

    Trung úy

  • Điều hành viên THCS
  • 773 Bài viết

Câu b bài hình chứng minh BP và CQ đi qua trung điểm EF
Gọi AK là đường kính của (O) và W là trung điểm BC.
Tam giác RBC và tam giác RFE đồng dạng
R, W, K thẳng hàng. Gọi T là trung điểm EF
Từ đó biến đổi góc như sau FRT=BRW=KRB=SAB=FRS nên RS đí qua T

Anh Khoa thi tốt không ạ :D 



#22
dogsteven

dogsteven

    Đại úy

  • Thành viên
  • 1567 Bài viết

câu 1b là tìm tất cả a mà :/

Tất cả a đều thoả

Quyết tâm off dài dài cày hình, số, tổ, rời rạc.


#23
IHateMath

IHateMath

    Thượng sĩ

  • Thành viên
  • 299 Bài viết

Em xin chém bài 4 
phần a thì  mò dễ k =3 
phần b ta cm k min =2017
Cách tô 
mỗi hàng tô 1 số cùng loại ( kể cả âm hay dương )
thì ta luôn có thoả vì lúc đó các số trên hàng k giao với cột
ta chỉ ra 1 cách mà bắt buộc k min =2017
xét bảng tô mà 2 đường chéo cạnh đường chéo lớn nhất tô trắng , 2 ô góc còn lại tô trắng và cả bảng đen
ta chỉ ra được là ô góc dưới và 2016 ô đường chéo lớn nhất k cùng mặt phẳng bờ AC với nó là các giá trị đôi 1 khác nhau 
=> k min =2017 
 

Bạn xem lại phần $b$. Đề bài nói rằng cách tô $k$-cân đối phải thỏa mãn điều kiện: Hai ô đối xứng qua đường chéo $AC$ phải điền cùng một số nguyên thuộc vào $[-k,k]$.

P/s: Xin lỗi bạn, mình hiểu nhầm ý bạn trong lời giải trên.


Bài viết đã được chỉnh sửa nội dung bởi IHateMath: 05-01-2017 - 14:28


#24
vda2000

vda2000

    Sĩ quan

  • Thành viên
  • 301 Bài viết

Tất cả a đều thoả

thực chất $a\geq\frac{-1}{10}$ để $u_2$ xác định nữa


$\boxed{\textrm{Silence is the peak of contempt!}}$

If you see this, you will visit my facebook.....!


#25
quangnamnvm

quangnamnvm

    Lính mới

  • Thành viên
  • 3 Bài viết

Câu 1:

Ý tưởng là để ý x = 3 là điểm nhạy cảm của dãy số.

a)Ta chứng minh các bước sau

- $x_n>3 \forall n$

- $|x_{n+1}-3|<\frac{|x_n-3|}{2}$

b)Chia làm hai TH:

Nếu tồn tại $n$ để $x_n>3$ thì cmtt như a)

Xét $x_n<3 \forall n$

Chọn dãy $v_n = \frac{1}{2} + \sqrt{2u_n+\frac{1}{4}}$, ta có $u_n>v_n$ và $lim v_n = 3$, suy ra $lim x_n = 3$ do bị kẹp

 

Câu 2:

Tồn tại

Chọn $A(x) = (x-1)^3-2$, $B(X) = x^2 + 2x - 4$, ta có:

$gcd(A,B) = 1$, $P(x) - x$ chia hết cho $A(x)$ và $Q(x) - (3x-1)$ chia hết cho $B(x)$

Suy ra, ta cần tìm $P(x)$ thoả mãn:

$P = A.Q + x = B.R + (3x-1)$

$\Leftrightarrow A.Q - B.R = 2x-1$

Vì $gcd(A,B) = 1$ nên theo thuật chia Euclid, tồn tại $Q,R$ hệ số nguyên thoả mãn, suy ra dpcm

Nếu có đa thức P(x). Ta đêm chia P(x) cho ((x - 1)^2 - 5)((x - 1)^3 - 2) ta được đa thức mới có bậc bé hơn 5 thỏa mãn yêu cầu bài toán. Nhưng nếu ta thử lại thì thấy không có? Hãy chỉ đa thức P(x) như thế thỏa mãn.



#26
vietdohoangtk7nqd

vietdohoangtk7nqd

    Hạ sĩ

  • Thành viên
  • 59 Bài viết

rốt cuộc câu 2 có tồn tại hay không mình mơ hồ quá



#27
dangkhuong

dangkhuong

    Sĩ quan

  • Thành viên
  • 312 Bài viết
Bài hình VMO ngày 1-mk vừa đánh xong mỏi cả tay :))
 
Lời giải: a) Ta thấy: $I$ là trung điểm của $AH$ nên $\angle IEH=\angle IHE=\angle AFE=\angle ACB=\angle HDB$(chú ý $AFHE,BFEC$ là các tứ giác nội tiếp) suy ra $IEDB$ nội tiếp. Chứng minh tương tự thì $IFDC$ nội tiếp. Ta để ý rằng: $I,F,E$ cùng nằm trên đường tròn $Euler$ của tam giác $ABC$ nên $M,N$ nằm trên trục đẳng phương của $(O)$ và đường tròn $Euler$ do đó chú ý tâm đường tròn $Euler$ là trung điểm $OH$ nên $OH\perp MN$(đpcm).
 
b) Trước tiên ta để ý rằng: $BP,CQ,EF$ đồng quy theo định lí $Pascal$ cho lục giác nội tiếp $RAPCDB$, ta chứng minh giao điểm $BP,CQ,EF$ là trung điểm $EF$. Gọi $S$ là chân đường cao hạ từ $A$ lên $BC$. Ta dễ thấy rằng: $S$ là trung điểm $HD$. Ta để ý thấy $\bigtriangleup FEB\sim \bigtriangleup HES(g.g)$. Gọi $J$ là trung điểm $EF$ thì: $\dfrac{EF}{HE}=\dfrac{BF}{HS}$ suy ra: $\dfrac{FJ}{HE}=\dfrac{BF}{HD}$ suy ra $\bigtriangleup JFB\sim \bigtriangleup DHE(c.g.c)$ vâyỵ $\angle ABJ=\angle HDE=\angle ABP$ nên $B,J,P$ thẳng hàng. Do đó $BP,CQ,EF$ đồng quy tại trung điểm $EF$. Ta để ý rằng: $AS,AH$ đẳng giác trong góc $\angle BAC$ nên $FHSE$ là 1 hình thang cân. Ta có kết quả quen thuộc là $R,H,L$ thẳng hàng($L$ là trung điểm $BC$) do đó chú ý rằng: $LF,LE$ là các tiếp tuyến đến $(AEF)$ nên $RFHE$ là 1 tứ giác điều hoà nên $S(RH,FE)=-1$ mà $SH\| EF$ nên theo tính chất hàng điểm điều hoà thì: $RS$ chia đôi $EF$. Vậy $RS,BP,CQ$ đồng quy.
 
P/s: Bài b) là mấy bài cộng dồn dame-trong đó có một bài là bổ đề của th Hùng đã viết bài trên Epsilon số tháng 6/2015 :))

Hình gửi kèm

  • hih132.png

:ukliam2:  :ukliam2:  :ukliam2:


#28
vietdohoangtk7nqd

vietdohoangtk7nqd

    Hạ sĩ

  • Thành viên
  • 59 Bài viết

cách mình làm giống cách của dangkhuong đấy



#29
lehakhiem212

lehakhiem212

    Trung sĩ

  • Thành viên
  • 113 Bài viết

 câu 3b: Cm được $BP, CQ$ đi qua trung điểm $EF$.

Gọi $M$ là trung điểm $BC$.Dễ chứng minh $R,H,M$ thẳng hàng.

Biến đổi góc cm được: $SOMR$ và $RIOM$ là các tứ giác nội tiếp.

Nên $IM,RS,EF$ đồng quy do là trục đẳng phương của 3 đường tròn (đường kính AH),(Euler của ABC),(RISM).

Mà $IM$ đi qua trung điểm $EF$. Ta có đpcm.



#30
Uchiha Sasuke 88

Uchiha Sasuke 88

    Binh nhì

  • Thành viên mới
  • 13 Bài viết

Mọi người đội em đều bảo bài 2 không tồn tại anh ạ



#31
Uchiha Sasuke 88

Uchiha Sasuke 88

    Binh nhì

  • Thành viên mới
  • 13 Bài viết

Bài 3b) em có dùng 1 bổ đề:

Cho tam giác ABC nội tiếp (O), M là trung điểm BC. đường cao BE,CF, trực tâm H. (AH) cắt (O) tại R Khi đó AR, EF,BC đồng quy và M,H,R thẳng hàng

Ý tưởng

Chứng minh BP, CQ đi qua trung điểm EF. Sau đó chứng minh RS đi qua trung điểm EF. dùng lượng giác suy ra cm tứ giác BDCR điều hòa. Dùng bổ đề suy ra RD là đối trung $\angle$BRC.

đpcm



#32
quangnamnvm

quangnamnvm

    Lính mới

  • Thành viên
  • 3 Bài viết

Câu 2: Theo mình

 

File gửi kèm


Bài viết đã được chỉnh sửa nội dung bởi quangnamnvm: 05-01-2017 - 18:40


#33
Superaceace

Superaceace

    Lính mới

  • Thành viên mới
  • 2 Bài viết

 

Câu 2:

Tồn tại

Chọn A(x)=(x1)32A(x)=(x−1)3−2B(X)=x2+2x4B(X)=x2+2x−4, ta có:

gcd(A,B)=1gcd(A,B)=1P(x)xP(x)−x chia hết cho A(x)A(x) và Q(x)(3x1)Q(x)−(3x−1) chia hết cho B(x)B(x)

Suy ra, ta cần tìm P(x)P(x) thoả mãn:

P=A.Q+x=B.R+(3x1)P=A.Q+x=B.R+(3x−1)

A.QB.R=2x1⇔A.Q−B.R=2x−1

Vì gcd(A,B)=1gcd(A,B)=1 nên theo thuật chia Euclid, tồn tại Q,RQ,R hệ số nguyên thoả mãn, suy ra dpcm

Q, R có chắc là hệ số nguyên không bạn



#34
SonKHTN1619

SonKHTN1619

    Binh nhất

  • Thành viên mới
  • 22 Bài viết

Giả sử tồn tại $P(x)$ thỏa mãn.

Do $x^3-2$ và $x^2-5$ lần lượt là các đa thức tối thiểu của $\sqrt[3]{2}$ và $\sqrt{5}$ nên ta có:

$x^3-2|P(x+1)-(x+1)$; (1)

$x^2-5|P(x+1)-(3x+2)$. (2)

Từ (1), tồn tại $Q(x)$ là đa thức hệ số nguyên sao cho $P(x+1)=(x+1)+(x^3-2)Q(x)$

Do đó,

(2)$\Leftrightarrow (5x-2)Q(x) \equiv 2x+1 (mod x^2-5)$

$\Leftrightarrow (5x-2)(5x+2)Q(x) \equiv (2x+1)(5x+2) (mod x^2-5)$

$\Leftrightarrow 121Q(x) \equiv 9x+52 (mod x^2-5)$

Với $x=-4, 0 \equiv 16 (mod 11)$, vô lý.

Do đó không tồn tại $P(x)$ thỏa mãn.

 


HSGS in my heart  :icon12:


#35
SonKHTN1619

SonKHTN1619

    Binh nhất

  • Thành viên mới
  • 22 Bài viết

Một mở rộng cho câu 3a, tuy nhiên ý tưởng là khá lộ liễu.
Cho $\Delta ABC$ nội tiếp $(O)$, $BE,CF$ là các đường cao, $H$ trực tâm. $AH$ cắt (O) tại điểm thứ hai $D$, $I$ trung điểm $AH$. $DB,DC$ cắt $EI,FI$ tại $M,N$. Gọi $S,T$ lần lượt là giao điểm thứ hai của $NH$ với $(BNE)$,$MH$ với $(CMF)$. CMR $(AH)$, đường tròn đối xứng $(O)$ qua $BC$, $(HST)$ đồng trục.


Bài viết đã được chỉnh sửa nội dung bởi SonKHTN1619: 05-01-2017 - 22:47

HSGS in my heart  :icon12:


#36
dangkhuong

dangkhuong

    Sĩ quan

  • Thành viên
  • 312 Bài viết

Một mở rộng cho câu 3a, tuy nhiên ý tưởng là khá lộ liễu.
Cho $\Delta ABC$ nội tiếp $(O)$, $BE,CF$ là các đường cao, $H$ trực tâm. $AH$ cắt (O) tại điểm thứ hai $D$, $I$ trung điểm $AH$. $DB,DC$ cắt $EI,FI$ tại $M,N$. Gọi $S,T$ lần lượt là giao điểm thứ hai của $NH$ với $(BNE)$,$MH$ với $(CMF)$. CMR $(AH)$, đường tròn đối xứng $(O)$ qua $BC$, $(HST)$ đồng trục.

Tại sao e không mở rộng kiểu thay đường cao thành đường tròn qua $B,C$ cắt $AB,AC$ tại $E,F$ nhỉ. Hình như bài toán chia đôi đoạn $EF$ vẫn đúng đấy.


:ukliam2:  :ukliam2:  :ukliam2:


#37
SonKHTN1619

SonKHTN1619

    Binh nhất

  • Thành viên mới
  • 22 Bài viết

Câu a hoàn toàn có thể viết lại bằng một đường tròn đi qua B,C như sau:

Gọi $\omega $ là đường tròn đi qua $B,C$ cắt $CA,AB$ tại $E, F$, tâm $K$. Gọi $L$ là tâm của $(AEF)$, $H$ là giao của $BE, CF$. Gọi $G$ là giao điểm của $KL$ với $AH$, $D$ là giao điểm thứ hai của $AH$ với $(O)$. $DB, DC$ cắt $EG,FG$ lần lượt tại $M,N$. CMR $OL \perp  MN$. 

Câu b em chưa biết nên mở rộng thế nào vì lúc này tuy tính chất chia đôi của $BP,CQ$ vẫn bảo toàn nhưng do $H$ không nằm trên $(AEF)$ nữa nên mất đi tính chia đôi của $RS$.


HSGS in my heart  :icon12:


#38
SonKHTN1619

SonKHTN1619

    Binh nhất

  • Thành viên mới
  • 22 Bài viết

VMO ngày 2

Hình gửi kèm

  • 15822721_1604055969609602_1427530542529003057_n.jpg

HSGS in my heart  :icon12:


#39
Dinh Xuan Hung

Dinh Xuan Hung

    Thành viên nổi bật 2015

  • Thành viên nổi bật 2016
  • 1396 Bài viết

   BỘ GIÁO DỤC VÀ ĐÀO TẠO                                KỲ THI CHỌN HỌC SINH GIỎI QUỐC GIA THPT NĂM 2017

                                                                                                         

       ĐỀ THI CHÍNH THỨC

            Môn Toán 

                         Thời gian : 180 phút

                                      Ngày thi thứ hai 06/01/2017

 

Bài 5 . (6,0 điểm).

 

Tìm tất cả các hàm số : $f:\mathbb{R}\rightarrow \mathbb{R}$ thỏa mãn hệ thức:

 

$$f\left ( xf\left ( y \right )-f\left ( x \right ) \right )=2f\left ( x \right )+xy$$

 

với mọi số thực $x,y$

 

Bài 6 . (7,0 điểm) 

 

Chứng minh rằng:

 

a)$\sum_{k=1}^{1008}kC_{2017}^{k}\equiv 0$ (mod $2017^2$ )

 

b)$\sum_{k=1}^{504}\left ( -1 \right )^kC_{2017}^{k}\equiv 3\left ( 2^{2016}-1 \right )$ (mod $2017^2$ )

 

Bài 7 . (7,0 điểm)

 

Cho tam giác nhọn $ABC$ nội tiếp đường tròn $(O)$  và $G$ là một điểm thuộc cung $BC$ không chứa $O$  của đường tròn $(I)$ ngoại tiếp tam giác $OBC$ . Đường tròn ngoại tiếp tam giác $ABG$ cắt $AC$ tại $E$ , đường tròn ngoại tiếp tam giác $ACG$ cắt $AB$ tại $F$ ( $E$ và $F$ khác $A$ )

 

a)Gọi $K$ là giao điểm của $BE$ và $CF$ . Chứng minh $AK,BC$ và $OG$ đồng quy

 

b)Cho $D$ là một điểm thuộc cung $\overbrace{BOC}$ chứa $O$ của đường tròn $(I)$ ; $GB$ cắt $CD$ tại $M$ . $GC$ cắt $BD$ tại $N$ . Giả sử $MN$ cắt $(O)$ tại hai điểm $P,Q$ .Chứng minh rằng: khi $G$ thay đổi trên cung BC không chứa $O$ của đường tròn $(I)$ , đường tròn ngoại tiếp $GPQ$ luôn đi qua hai điểm cố định

 

HẾT


Bài viết đã được chỉnh sửa nội dung bởi Dinh Xuan Hung: 06-01-2017 - 11:50


#40
quangkhaiolympic

quangkhaiolympic

    Lính mới

  • Thành viên mới
  • 5 Bài viết

mình làm hết ngày 2 nè he he :))






2 người đang xem chủ đề

0 thành viên, 2 khách, 0 thành viên ẩn danh